Đến nội dung

Hình ảnh

/ $$\frac{2\sqrt{a}}{a^{3}+b^{2}}+\frac{2\sqrt{b}}{b^{3}+c^{2}}+\frac{2\sqrt{c}}{c^{3}+a^{2}} \leq \frac{1}{a^{2}}+\frac{1}{b^{2}}+\frac{1}{c^{2}}$$


  • Please log in to reply
Chủ đề này có 5 trả lời

#1
xuanhung

xuanhung

    Binh nhất

  • Thành viên
  • 38 Bài viết

Giúp mình vài bài này với: Cho a,b,c>0, CMR:

1/ $$\frac{a^{8}+b^{8}+c^{8}}{a^{3}b^{3}c^{3}}\geqslant \frac{1}{a}+\frac{1}{b}+\frac{1}{c}$$

 

2/ $$\frac{a^{2}+b^{2}}{c}+\frac{b^{2}+c^{2}}{a}+\frac{c^{2}+a^{2}}{b}\geq 2(a+b+c)$$

 

3/ $$\frac{2\sqrt{a}}{a^{3}+b^{2}}+\frac{2\sqrt{b}}{b^{3}+c^{2}}+\frac{2\sqrt{c}}{c^{3}+a^{2}} \leq \frac{1}{a^{2}}+\frac{1}{b^{2}}+\frac{1}{c^{2}}$$


Bài viết đã được chỉnh sửa nội dung bởi xuanhung: 15-06-2013 - 16:43

Doesn't mean the all

Doesn't mean nothing

Doesn't mean the best

Doesn't mean the worst


#2
trauvang97

trauvang97

    Sĩ quan

  • Thành viên
  • 402 Bài viết

Giúp mình vài bài này với: Cho a,b,c>0, CMR:

2/ $$\frac{a^{2}+b^{2}}{c}+\frac{b^{2}+c^{2}}{a}+\frac{c^{2}+a^{2}}{b}\geq 2(a+b+c)$$

 

3/ $$\frac{2\sqrt{a}}{a^{3}+b^{2}}+\frac{2\sqrt{b}}{b^{3}+c^{2}}+\frac{2\sqrt{c}}{c^{3}+a^{2}} \leq \frac{1}{a^{2}}+\frac{1}{b^{2}}+\frac{1}{c^{2}}$$

 

2/ 

 

Đặt vế trái của biểu thức đã cho là $A$

 

Áp dụng bất đẳng thức Cauchy ta có: $A\geq 2\left ( \frac{ab}{c}+\frac{bc}{a}+\frac{ca}{b} \right )$

 

Do đó bất đẳng thức cần chứng minh tương đương với:

 

       $\frac{ab}{c}+\frac{bc}{a}+\frac{ca}{b}\geq a+b+c$

 

Áp dụng bất đẳng thức Cauchy cho 3 bộ 2 số $\left ( \frac{ab}{c},\frac{bc}{a} \right );\left ( \frac{bc}{a},\frac{ca}{b} \right );\left ( \frac{ca}{b},\frac{ab}{c} \right )$ ta có đpcm

 

3/ Áp dụng bất đẳng thức Cauchy cho các mẫu thức ta có: $a^{3}+b^{2}\geq 2ab\sqrt{a}$

 

Tương tự: $b^{3}+c^{2}\geq 2bc\sqrt{b}$, $c^{3}+a^{2}\geq 2ca\sqrt{c}$

 

Do đó bất đẳng thức cần chứng minh tương đương với bất đẳng thức:

 

   $\frac{1}{ab}+\frac{1}{bc}+\frac{1}{ca}\leq \frac{1}{a^{2}}+\frac{1}{b^{2}}+\frac{1}{c^{2}}$ luôn đúng nên ta có đpcm


Bài viết đã được chỉnh sửa nội dung bởi trauvang97: 15-06-2013 - 16:49


#3
Peter97

Peter97

    Binh nhất

  • Thành viên
  • 45 Bài viết

2/ $$\frac{a^{2}+b^{2}}{c}+\frac{b^{2}+c^{2}}{a}+\frac{c^{2}+a^{2}}{b}\geq 2(a+b+c)$$

 

 

$\Leftrightarrow \frac{a^{2}}{c} + \frac{c^{2}}{a} + \frac{b^{2}}{c} + \frac{c^{2}}{b} + \frac{a^{2}}{b} + \frac{b^{2}}{a} \geq 2(a + b + c )$

Ta có $a^{3} + b^{3} \geq ab(a + b) \Leftrightarrow (a + b)(a - b)^{2}\geq 0$ ( BĐT đúng) 

 

Áp dụng ta có :$\frac{a^{3} + c^{3}}{ac} \geq a + c$ Tương tự  rồi cộng vế vs vế ta đc dpcm 

Dấu bằng xáy ra khi a = b = c


EM YÊU BÁC HỒ..... :oto:


#4
Peter97

Peter97

    Binh nhất

  • Thành viên
  • 45 Bài viết

Giúp mình vài bài này với: Cho a,b,c>0, CMR:

1/ $$\frac{a^{8}+b^{8}+c^{8}}{a^{3}b^{3}c^{3}}\geqslant \frac{1}{a}+\frac{1}{b}+\frac{1}{c}$

Dễ thấy BĐT $a^{2} + b^{2} + c^{2} \geq ab + ac+ bc$ luôn đúng

Áp dụng 

BĐT trên $\Leftrightarrow a^{8} + b^{8} + c^{8} \geq a^{2}b^{2}c^{2}(ab + ac + bc)$

Ta có:

$a^{8} + b^{8} + c^{8} \geq a^{4}b^{4} + b^{4}c^{4} + a^{4}c^{4} \geq a^{2}b^{4}c^{2} + a^{2}b^{2}c^{4} + a^{4}b^{2}c^{2} \geq a^{2}b^{2}c^{2}(a^{2} + b^{2} + c^{2})\geq a^{2}b^{2}c^{2}(ab+ ac+ bc)$ 


EM YÊU BÁC HỒ..... :oto:


#5
Zaraki

Zaraki

    PQT

  • Phó Quản lý Toán Cao cấp
  • 4273 Bài viết

Dễ thấy BĐT $a^{2} + b^{2} + c^{2} \geq ab + ac+ bc$ luôn đúng

Áp dụng 

BĐT trên $\Leftrightarrow a^{8} + b^{8} + c^{8} \geq a^{2}b^{2}c^{2}(ab + ac + bc)$

Ta có:

$a^{8} + b^{8} + c^{8} \geq a^{4}b^{4} + b^{4}c^{4} + a^{4}c^{4} \geq a^{2}b^{4}c^{2} + a^{2}b^{2}c^{4} + a^{4}b^{2}c^{2} \geq a^{2}b^{2}c^{2}(a^{2} + b^{2} + c^{2})\geq a^{2}b^{2}c^{2}(ab+ ac+ bc)$ 

Cách khác. Bất đẳng thức tương đương với việc chứng minh $a^8+b^8+c^8 \ge a^2b^3c^3+b^2c^3a^3+c^2a^3b^3$.

Áp dụng BĐT AM-GM ta có $2a^8+3b^8+3c^8 \ge 8a^2b^3c^3$.

Tương tự rồi cộng lại, ta có đpcm.


Bài viết đã được chỉnh sửa nội dung bởi Jinbe: 15-06-2013 - 17:13

Discovery is a child’s privilege. I mean the small child, the child who is not afraid to be wrong, to look silly, to not be serious, and to act differently from everyone else. He is also not afraid that the things he is interested in are in bad taste or turn out to be different from his expectations, from what they should be, or rather he is not afraid of what they actually are. He ignores the silent and flawless consensus that is part of the air we breathe – the consensus of all the people who are, or are reputed to be, reasonable.

 

Grothendieck, Récoltes et Semailles (“Crops and Seeds”). 


#6
25 minutes

25 minutes

    Thành viên nổi bật 2015

  • Hiệp sỹ
  • 2795 Bài viết

1/ $$\frac{a^{8}+b^{8}+c^{8}}{a^{3}b^{3}c^{3}}\geqslant \frac{1}{a}+\frac{1}{b}+\frac{1}{c}$$

Cách khác : 

BĐT $\Leftrightarrow \frac{a^5}{b^3c^3}+\frac{b^5}{c^3a^3}+\frac{c^5}{a^3b^3} \geq \frac{1}{a}+\frac{1}{b}+\frac{1}{c}$

Áp dụng AM-GM ta có $\frac{a^5}{b^3c^3}+\frac{1}{a}+\frac{1}{b} \geq \frac{3}{bc}$

Tương tự 2 bđt còn lại rồi cộng vào ta có 

                        $\sum \frac{a^5}{b^3c^3}+\sum \frac{2}{a} \geq \sum \frac{3}{bc}$  (1)

Dễ thấy $\sum \frac{3}{bc} \geq 3( \frac{1}{a}+\frac{1}{b}+\frac{1}{c})$ theo AM-GM (2)

Từ (1) và (2) ta có đpcm


Hãy theo đuổi đam mê, thành công sẽ theo đuổi bạn.



Thảo luận BĐT ôn thi Đại học tại đây





0 người đang xem chủ đề

0 thành viên, 0 khách, 0 thành viên ẩn danh